Difference between revisions of "2021 Fall AMC 12A Problems/Problem 4"
(added solution 2) |
(→Solution 2) |
||
(15 intermediate revisions by 7 users not shown) | |||
Line 6: | Line 6: | ||
<math>\textbf{(A)}\ 1 \qquad\textbf{(B)}\ 3 \qquad\textbf{(C)}\ 5 \qquad\textbf{(D)}\ 7 \qquad\textbf{(E)}\ 9</math> | <math>\textbf{(A)}\ 1 \qquad\textbf{(B)}\ 3 \qquad\textbf{(C)}\ 5 \qquad\textbf{(D)}\ 7 \qquad\textbf{(E)}\ 9</math> | ||
− | == Solution == | + | == Solution 1== |
First, modulo <math>2</math> or <math>5</math>, <math>\underline{20210A} \equiv A</math>. | First, modulo <math>2</math> or <math>5</math>, <math>\underline{20210A} \equiv A</math>. | ||
Hence, <math>A \neq 0, 2, 4, 5, 6, 8</math>. | Hence, <math>A \neq 0, 2, 4, 5, 6, 8</math>. | ||
Line 16: | Line 16: | ||
Hence, <math>A \neq 3</math>. | Hence, <math>A \neq 3</math>. | ||
− | Therefore, the answer is <math>\boxed{\textbf{(E) }9}</math>. | + | Therefore, the answer is <math>\boxed{\textbf{(E)}\ 9}</math>. |
− | ~NH14 | + | ~NH14 ~Steven Chen (www.professorchenedu.com) |
− | + | ==Solution 2 (Elimination)== | |
+ | Any number ending in <math>5</math> is divisible by <math>5</math>. So we can eliminate option <math>\textbf{(C)}</math>. | ||
− | ==Solution | + | If the sum of the digits of a number is divisible by <math>3</math>, the number is divisible by <math>3</math>. The sum of the digits of this number is <math>2 + 0 + 2 + 1 + 0 + A = 5 + A</math>. If <math>5 + A</math> is divisible by <math>3</math>, the number is divisible by <math>3</math>. Thus we can eliminate options <math>\textbf{(A)}</math> and <math>\textbf{(D)}</math>. |
+ | |||
+ | So the correct option is either <math>\textbf{(B)}</math> or <math>\textbf{(E)}</math>. Let's try dividing the number with some integers. | ||
+ | |||
+ | <math>20210A/7 = 2887x</math>, where <math>x</math> is <math>1A/7</math>. Since <math>13</math> and <math>19</math> are both indivisible by <math>7</math>, this does not help us narrow the choices down. | ||
+ | |||
+ | <math>20210A/11 = 1837x</math>, where <math>x</math> is <math>3A/11</math>. Since <math>33/11 = 3</math>, option <math>\textbf{(B)}</math> would make <math>20210A</math> divisible by <math>11</math>. Thus, by elimination, the correct choice must be option <math>\boxed{\textbf{(E)}\ 9}</math>. | ||
+ | |||
+ | ~ZoBro23 | ||
+ | |||
+ | ==Solution 3== | ||
<math>202100 \implies</math> divisible by <math>2</math>. | <math>202100 \implies</math> divisible by <math>2</math>. | ||
Line 41: | Line 52: | ||
<math>202108 \implies</math> divisible by <math>2</math>. | <math>202108 \implies</math> divisible by <math>2</math>. | ||
− | This leaves only <math>A=\boxed{\textbf{(E) }9}</math>. | + | This leaves only <math>A=\boxed{\textbf{(E)}\ 9}</math>. |
~wamofan | ~wamofan | ||
+ | |||
+ | ==Video Solution (Simple and Quick)== | ||
+ | https://youtu.be/7_Dg9b2hQ5U | ||
+ | |||
+ | ~Education, the Study of Everything | ||
+ | |||
+ | ==Video Solution== | ||
+ | https://youtu.be/jxnTkY3eb5Y | ||
+ | |||
+ | ~savannahsolver | ||
+ | |||
+ | ==Video Solution== | ||
+ | https://youtu.be/AgzDyKlmNAo | ||
+ | |||
+ | ~Charles3829 | ||
+ | |||
+ | ==Video Solution by TheBeautyofMath== | ||
+ | for AMC 10: https://youtu.be/o98vGHAUYjM?t=623 | ||
+ | |||
+ | for AMC 12: https://youtu.be/jY-17W6dA3c?t=392 | ||
+ | |||
+ | ~IceMatrix | ||
+ | |||
+ | ==Video Solution== | ||
+ | https://youtu.be/7TnFYSJ8i14 | ||
+ | |||
+ | ~Lucas | ||
+ | |||
+ | ==Video Solution== | ||
+ | https://youtu.be/7_Dg9b2hQ5U | ||
+ | |||
+ | ~Education, the Study of Everything | ||
==See Also== | ==See Also== |
Latest revision as of 20:13, 12 July 2023
- The following problem is from both the 2021 Fall AMC 10A #5 and 2021 Fall AMC 12A #4, so both problems redirect to this page.
Contents
Problem
The six-digit number is prime for only one digit What is
Solution 1
First, modulo or , . Hence, .
Second modulo , . Hence, .
Third, modulo , . Hence, .
Therefore, the answer is .
~NH14 ~Steven Chen (www.professorchenedu.com)
Solution 2 (Elimination)
Any number ending in is divisible by . So we can eliminate option .
If the sum of the digits of a number is divisible by , the number is divisible by . The sum of the digits of this number is . If is divisible by , the number is divisible by . Thus we can eliminate options and .
So the correct option is either or . Let's try dividing the number with some integers.
, where is . Since and are both indivisible by , this does not help us narrow the choices down.
, where is . Since , option would make divisible by . Thus, by elimination, the correct choice must be option .
~ZoBro23
Solution 3
divisible by .
divisible by .
divisible by .
divisible by .
divisible by .
divisible by .
divisible by .
divisible by .
divisible by .
This leaves only .
~wamofan
Video Solution (Simple and Quick)
~Education, the Study of Everything
Video Solution
~savannahsolver
Video Solution
~Charles3829
Video Solution by TheBeautyofMath
for AMC 10: https://youtu.be/o98vGHAUYjM?t=623
for AMC 12: https://youtu.be/jY-17W6dA3c?t=392
~IceMatrix
Video Solution
~Lucas
Video Solution
~Education, the Study of Everything
See Also
2021 Fall AMC 12A (Problems • Answer Key • Resources) | |
Preceded by Problem 3 |
Followed by Problem 5 |
1 • 2 • 3 • 4 • 5 • 6 • 7 • 8 • 9 • 10 • 11 • 12 • 13 • 14 • 15 • 16 • 17 • 18 • 19 • 20 • 21 • 22 • 23 • 24 • 25 | |
All AMC 12 Problems and Solutions |
2021 Fall AMC 10A (Problems • Answer Key • Resources) | ||
Preceded by Problem 4 |
Followed by Problem 6 | |
1 • 2 • 3 • 4 • 5 • 6 • 7 • 8 • 9 • 10 • 11 • 12 • 13 • 14 • 15 • 16 • 17 • 18 • 19 • 20 • 21 • 22 • 23 • 24 • 25 | ||
All AMC 10 Problems and Solutions |
The problems on this page are copyrighted by the Mathematical Association of America's American Mathematics Competitions.